LSAT and Law School Admissions Forum

Get expert LSAT preparation and law school admissions advice from PowerScore Test Preparation.

User avatar
 lavalsat
  • Posts: 13
  • Joined: Jan 26, 2021
|
#84325
Hello,

I am wondering if someone could help me understand a certain piece of this question. Would't the assumption that the argument depends on be that "The per capita GDP of the EEC was not lower by $1,000 or more in 1990 than it had been in 1980?

If I am understanding correctly- the assumption, as stated in answer D, leaves open that possibility that the decrease of EEC GDP was EXACTLY $1,000 and therefore the standard of living in County A could have remained EXACTLY the same. Then this assumption as stated in answer D wouldn't necessarily be what the argument depends on, it would just strengthen the argument.

I am having trouble fully getting on board saying that the argument "depends" on this assumption. This one has me particularly confused!
User avatar
 lavalsat
  • Posts: 13
  • Joined: Jan 26, 2021
|
#84326
In regards to my above comment, if I am correct that answer D leaves open the possibility that County A standard of living could have remained exactly the same, since this question asks for an assumption the argument depends on, is this answer choice technically flawed in its reasoning? If this answer choice is flawed (even if it is ever so slightly) why wasn't this addressed by LSAC? I would have much less of a problem if we were looking for an answer that supported the argument instead of one on which the argument depended on.

I guess my final two questions are:
1) Is the answer in choice D flawed?
2) If the answer is flawed, why did LSAC not address it or toss this question out?
 Adam Tyson
PowerScore Staff
  • PowerScore Staff
  • Posts: 5153
  • Joined: Apr 14, 2011
|
#84356
Good question, lavalsat! If this was a Justify the Conclusion question, in which we needed to prove the argument was correct, answer D would indeed be insufficient, because we would also need to eliminate the possibility that EEC GDP did not drop by exactly $1000. But since this is an Assumption question, the correct answer doesn't need to prove the argument is correct. Rather, the argument has to prove the answer is correct!

There can be many assumptions in one argument. Here, the author has to make at least two:

1) EEC GDP did not drop by exactly $1000
2) EEC GDP did not drop by MORE than $1000

Both of those assumptions are necessary, although neither, by itself, is sufficient. The question doesn't ask for ALL the required assumptions, but just for one such assumption. Answer D is required, because if it was not true the argument would fall apart completely.

Here's a quick analogy:

I have never broken my leg. Therefore, I have never broken a limb.

What must I have assumed?

1) I have never broken my right arm
2) I have never broken my left arm
3) I have never broken some limb other than a leg

Answer 1 is a required assumption, even if it is not "complete" in the sense that there are still other required assumptions for the argument to be good. There's nothing wrong with that answer! The proof is that if I negate it - I HAVE broken my right arm - the argument turns out to be nonsense.

Be sure you aren't mixing up an Assumption question stem (aka Necessary Assumption) for a Justify the Conclusion stem (aka Sufficient Assumption). That may be where you got confused on this one.
User avatar
 Camhaynes9
  • Posts: 1
  • Joined: Jan 31, 2023
|
#99076
Hi,

My confusion with this question is when the answer says more then $1000. If it were to drop by exactly 1000$. I know I’m missing something somewhere but to me this allows for the possibility that the per capita GDP drops by exactly 1000 making there be a 1000$ difference, and country A not having a better quality of life on avg because it stays the same. If it drops 1001$ then A must drop 1$ and if it drops 999$ then A must increase 1$ but if it drops 1000$ A stays the same and this makes this assumption incorrect as the GDP was not lower by more then 1000 and the quality of life in A does not increase.

I know I’m off somewhere but I was hoping for some guidance in where I am off.
User avatar
 Jeff Wren
PowerScore Staff
  • PowerScore Staff
  • Posts: 385
  • Joined: Oct 19, 2022
|
#99092
Hi Cam,

I think that what is causing the confusion is understanding exactly what necessary assumption questions (which we just call Assumption questions) are asking and how they differ from sufficient assumption questions (which we call Justify questions). This confusion is very common among test takers and is one of the reasons that Assumption questions are often the most difficult question type for most test takers.

For a necessary assumption, the answer does not have to Prove the conclusion of the argument. Instead, what you need to ask when examining each answer choice is "Is this statement something that the person making the argument must believe in order for the argument to make sense."

For example, if I claim "My friend John is a great tennis player,"

An acceptable answer to a necessary assumption to this claim would be:

"John can hit the ball over the net."

Now just being able to hit the ball over the net is a long way from being a great tennis play. This definitely doesn't prove that John is a great tennis player and would therefore be a wrong answer in a Justify question.

But he question here isn't asking for an answer that proves the conclusion, just something necessary for the conclusion.

If you were to ask me "Can John hit the ball over the net?" I'd reply "Of course! That's so obvious that I didn't even bother mentioning it."

Similarly, in this question, the argument requires that "the per capita GDP of the European Economic Community was not lower by more than $1,000 in 1990 than it had been in 1980." If that statement were untrue, i.e. if "the per capita GDP of the European Economic Community was lower by more than $1,000 in 1990 than it had been in 1980," then the argument completely falls apart. The fact that this answer still allows the possibility that the per capita GDP of the EEC was lowered by exactly $1000 (in which case the conclusion would not be true) does not make it a wrong answer in the same way that "John being able to hit the ball over the net" still allows the possibility that he is not a great tennis player.

This is why using our Assumption Negation Technique (discussed in Lesson 5 of our course and the LR Bible) can be so helpful in handling these questions.
 madelineunruh01
  • Posts: 4
  • Joined: Feb 11, 2024
|
#105427
The wording of answer choice D tripped me up here. I think I would have gotten the question correct if answer choice D had said "the per capita GDP of the EEC was not lower by $1,000". But, since it said "lower than $1,000" that number then felt irrelevant to the stimulus. How do I reconcile the difference in language? It is so small that I want to say its a fluke and move on but I've learned to never ignore any singular word in an LSAT question because sometimes it makes or breaks the question...here it definitely broke it haha. Thanks!
 madelineunruh01
  • Posts: 4
  • Joined: Feb 11, 2024
|
#105428
madelineunruh01 wrote: Thu Feb 22, 2024 8:28 pm The wording of answer choice D tripped me up here. I think I would have gotten the question correct if answer choice D had said "the per capita GDP of the EEC was not lower by $1,000". But, since it said "lower than $1,000" that number then felt irrelevant to the stimulus. How do I reconcile the difference in language? It is so small that I want to say its a fluke and move on but I've learned to never ignore any singular word in an LSAT question because sometimes it makes or breaks the question...here it definitely broke it haha. Thanks!
I can't edit my post but there is a typo. The answer choice reads "lower by MORE than 1,000" and not "lower than 1,000" as I incorrectly typed.
 Luke Haqq
PowerScore Staff
  • PowerScore Staff
  • Posts: 722
  • Joined: Apr 26, 2012
|
#105445
Hi madelineunruh01!

Since this is an assumption question, consider using the Assumption Negation technique to see how the wording of (D) applies to the stimulus. That involves negating the answer choice and plugging the result back into the stimulus; if this makes the argument fall apart, then it is an assumption on which the argument depends.

Here, the negation of (D) would be "The per capita GDP of the European Economic Community was [...] lower by more than $1,000 in 1990 than it had been in 1980." Note in the stimulus that the fact about Country A is defined in relation to the average GDP of the European Economic Community. The stimulus seems to be making an assumption that, because the difference had grown (by $1,000) between Country A and the average GDP of the European Economic Community from 1980 to 1990, the average standard of living must be higher. However, what if the European Economic Community's average GDP had lowered over that decade? If it had lowered by, say, $2,000, then the argument falls apart. It would not follow from the premises that the average standard of living is higher in Country A, because the greater difference in GDPs would instead be caused by a fall in GDP and standard of living.

This is what answer choice (D) gets at, and if the European Economic Community's average GDP had lowered by $1,001, the same reasoning would apply. That also seems to be true even if the amount were $1,000, but as Adam notes above, there can be multiple assumptions made by a stimulus; the task is to identify just one.

Get the most out of your LSAT Prep Plus subscription.

Analyze and track your performance with our Testing and Analytics Package.